[Orbital angular momentum operator] I am not able to get to the right part in B.30
πŸ‘︎ 12
πŸ’¬︎
πŸ‘€︎ u/linlithgowgoback
πŸ“…︎ Oct 17 2021
🚨︎ report
Checking if ψ is an eigenfunction of the operator for the z-component of angular momentum?

I'm working through a textbook I found on quantum mechanics and I'm having some difficulty with the problem sets. I am not a student, I'm just interested in physics, and I figure I can't really say I understand it to any degree if I can't do the math. I've reached out to the publisher for a solutions manual but they won't provide it because I'm not an instructor, which I understand but is still annoying.

Here is the problem (please excuse any formatting issues):

The operator for the z-component of angular momentum is given by:

Lz = -iℏ (βˆ‚/βˆ‚Ο†)

You're asked to determine whether or not these functions are eigenfunctions of the operator:

  1. ψ = sin(Ο†) e^(iΟ†)
  2. ψ = sin^k(Ο†) e^(ikΟ†), k=constant
  3. ψ = sin(Ο†) e^(-iΟ†)

When I work the problems, it doesn't look like ANY of them are eigenfunctions of Lz, but I'm sure that isn't right, because you're supposed to use the results for later problems. Here is my work:

  1. Lz ψ = -iℏ (βˆ‚/βˆ‚Ο†) sin(Ο†) e^(iΟ†)

= -iℏ [cos(Ο†) e^(iΟ†) + i sin(Ο†) e^(iΟ†)]

= -iℏ [cos(Ο†) + i sin(Ο†)] e^(iΟ†)

= -iℏ e^(i2Ο†)

(or expressing it in terms of ψ...)

= [-iℏ e^(iΟ†) / sin(Ο†)] ψ

  1. Lz ψ = -iℏ (βˆ‚/βˆ‚Ο†) sin^k(Ο†) e^(ikΟ†)

= -iℏ [k sin^(k-1)(Ο†) cos(Ο†) e^(ikΟ†) + ik sin^k(Ο†) e^(ikΟ†)]

= -iℏ [k sin^(k-1)(Ο†) e^(ikΟ†)] [cos(Ο†) + i sin(Ο†)]

= -iℏ [k sin^(k-1)(Ο†) e^(i2kΟ†)]

= [-iℏ k e^(ikΟ†) / sin(Ο†)] ψ

  1. Lz ψ = -iℏ (βˆ‚/βˆ‚Ο†) sin(Ο†) e^(-iΟ†)

= -iℏ [cos(Ο†) e^(-iΟ†) - i sin(Ο†) e^(-iΟ†)]

= -iℏ [cos(Ο†) - i sin(Ο†)] e^(-iΟ†)

= -iℏ e^(-i2Ο†)

= [-iℏ e^(-iΟ†) / sin(Ο†)] ψ

None of the last quantities in brackets look like constants to me, so they can't be solutions to Lz ψ = β„“ ψ, for some constant β„“, right? Meaning they aren't eigenfunctions of Lz, right? But maybe my math is bad, maybe I'm overlooking some identity, maybe I'm failing to understand something fundamental...This is an early chapter problem, so I'm pretty discouraged about my ability to ever grasp this material. I've lost momentum and am just bogged down in a frustrating math puzzle, one that probably any undergraduate could solve easily...In any case, I would appreciate some assistance.

πŸ‘︎ 13
πŸ’¬︎
πŸ‘€︎ u/BurnishChewie
πŸ“…︎ Sep 01 2021
🚨︎ report
Angular Momentum Operator Equation (I think?)

https://cdn.discordapp.com/attachments/743934084560060517/883881586939006996/unknown.png

I was presented with this question with zero context or clarification and no way to get any.

I'm not so much looking for a solution, moreso if anyone can parse what the question is actually asking/what tools does someone usually use to solve this?

From googling I've found that J_(-1)/J_(0) appears to be a ratio of angular momentum operators (https://en.wikipedia.org/wiki/Ladder_operator#Applications_in_atomic_and_molecular_physics), and it's being equated to some generic complex number. Am I trying to do a Taylor expansion or something? I'm not sure why the equation has i - sqrt(-1) either since that's just zero. It seems more like a statement than a question so I'm generally confused on where to go from here.

πŸ‘︎ 11
πŸ’¬︎
πŸ‘€︎ u/PurpleSatire
πŸ“…︎ Sep 05 2021
🚨︎ report
Is there an eigenstate to the angular momentum ladder operator?

Is there something similar to the glaube state as left and right hand eigenstate of the destruction and creaton operator of the harmonic qiantum oscillator?

πŸ‘︎ 3
πŸ’¬︎
πŸ‘€︎ u/KippieDaoud
πŸ“…︎ Jun 21 2018
🚨︎ report
Why did the angular momentum operators work from home

They don't commute

πŸ‘︎ 43
πŸ’¬︎
πŸ‘€︎ u/AltoidNerd
πŸ“…︎ Oct 20 2015
🚨︎ report
In the spherical coordinates representation of the L^2 angular momentum operator, why are we allowed to have (sin theta) in the denominator(s)?

http://en.wikipedia.org/wiki/Angular_momentum_operator

At the bottom of the Wikipedia page is the formula for the angular momentum L squared operator in spherical coordinates.

Isn't this operator undefined when theta is 0 (and thus sin theta is 0)?

πŸ‘︎ 9
πŸ’¬︎
πŸ‘€︎ u/260
πŸ“…︎ May 11 2015
🚨︎ report
The total angular momentum operator in the coupled basis

I'm having trouble with this simple concept I thought I understood.

the total angular momentum is supposed to be just that. If I have two particles, each spin 1/2, then the total angular momentum (ignoring orbital angular momentum) is 1, right? Yet my book wants to tell me that 0 is also an option.

If j1=1/2, and j2=1/2,

My book uses F for the total a.m.. How can F ever be anything but 1? The book says it can go as low as j1-j2, but if F is the total angular momentum, and the j's never change, then how can the total j1+j2 change?

It would seem that it has something to do with linear combinations of uncoupled states, such as 1/sqrt2(|j1j2m1m2> - |j1j2m1m2>), but it still makes no sense to me because the other superposition state 1/sqrt2(|j1j2m1m2> + |j1j2m1m2>) does not yield F=2/sqrt2.

πŸ‘︎ 3
πŸ’¬︎
πŸ‘€︎ u/Konijndijk
πŸ“…︎ Feb 17 2015
🚨︎ report
A teacher demonstrates angular momentum. v.redd.it/kzdedpf04j981
πŸ‘︎ 3k
πŸ’¬︎
πŸ‘€︎ u/Emigoooo
πŸ“…︎ Jan 03 2022
🚨︎ report
Is the total angular momentum ladder operator equivalent to the sum of those for the orbital and spin angular momenta?

Can the total angular momentum ladder operator be expressed as [; \hat{J}\pm = \hat{L}\pm + \hat{S}_\pm ;]?

The problem in question is here. I believe I can do this by expressing the total angular momentum state as a superposition of [; |l=1, m_l = -1, s = 1/2, m_s = 1/2> ;] and [; |l=1, m_l = 0, s = 1/2, m_s = -1/2> ;]. Then using the [;\hat{J}_\pm ;] operators I should be able to get enough information to normalise the state. I'm not sure how I can apply these operators to the linear superposition though.

πŸ‘︎ 2
πŸ’¬︎
πŸ‘€︎ u/ModeHopper
πŸ“…︎ Apr 19 2018
🚨︎ report
Where to the ladder operators for the orbital angular momentum eigenstates come from?

Hey, everyone. This is my first post on reddit. I'm trained as an engineer, but I've been slowly teaching myself QM through Griffith's book in conjunction with Atkin's Physical Chemistry. QM introduced me to the ladder operator techniques in order to get different eigenstates for the quantum harmonic oscillator and the angular momentum problems.

I understand that in the quantum harmonic oscillator problem, the raising and lowering operators come from an attempt to factor the Hamiltonian (Griffith, pg. 42):

H * psi = E * psi 1/2m * [p^2 + (mwx)^2] = E * psi

Where p is the momentum operator and x is the position operator. An attempt to factor [p^2 + (mwx)^2] yields:

a+ = constant * (i * p + mwx)

a- = constant * (i * p - mwx)

These are the raising and lowering ladder operators for the quantum harmonic oscillator and can be used to represent the Hamiltonian as:

H = h_bar * w * (a+a- + 1/2)

H = h_bar * w * (a-a+ - 1/2)

However, when we get to angular momentum, every single source that I have come across simply defines the raising and lowering operators as:

L+ = Lx + iLy

L- = Lx - iLy

without any particular reason as to how they got to these ladder operators. If I were taking a course at a university, I would simply ask the professor, but unfortunately I do not have that resource available right now. So, I want to ask what is the rationale for defining these two ladder operators this way?


My attempt at a solution to this would be to take into account the angular momentum operators L^2 and Lz. Both operators are hermitian and therefore give real values. L^2 can also be defined as:

L^2 = Lx^2 + Ly^2 + Lz^2

Since it is established that Lz is hermitian, Lz^2 must be nonzero and real. Therefore:

[L^2 - Lz^2] * psi = some real value = [Lx^2 + Ly^2] * psi

Factoring [Lx^2 + Ly^2] leads to:

L+ = Lx + iLy

L- = Lx - iLy

In an attempt to find an eigenstate that will satisfy both L^2 and Lz simultaneously. Am I on the right track here?


edit: Edited for weird symbol stuff.

πŸ‘︎ 4
πŸ’¬︎
πŸ‘€︎ u/Red_Horizon
πŸ“…︎ Feb 26 2015
🚨︎ report
Self balancing cube by conservation of angular momentum. This is how satellites are able to orient themselves without engines. gfycat.com/plainshabbycoc…
πŸ‘︎ 3k
πŸ’¬︎
πŸ‘€︎ u/juntawflo
πŸ“…︎ Dec 05 2021
🚨︎ report
Angular momentum demo - Arms IN vs OUT v.redd.it/38nog119sj581
πŸ‘︎ 1k
πŸ’¬︎
πŸ‘€︎ u/highnchillin_
πŸ“…︎ Dec 14 2021
🚨︎ report
A teacher demonstrates angular momentum. v.redd.it/h6c7k1tt3j981
πŸ‘︎ 181
πŸ’¬︎
πŸ‘€︎ u/Emigoooo
πŸ“…︎ Jan 03 2022
🚨︎ report
Angular momentum v.redd.it/zia9j6oa39w71
πŸ‘︎ 16k
πŸ’¬︎
πŸ‘€︎ u/Palana
πŸ“…︎ Oct 28 2021
🚨︎ report
If angular momentum was as intuitive as linear momentum :(
πŸ‘︎ 103
πŸ’¬︎
πŸ‘€︎ u/AB_001
πŸ“…︎ Jan 07 2022
🚨︎ report
Self balancing cube using the conservation of angular momentum v.redd.it/pwop13lyyq381
πŸ‘︎ 932
πŸ’¬︎
πŸ‘€︎ u/Einstein7-
πŸ“…︎ Dec 05 2021
🚨︎ report
rotation matrix eigenvalues/eigenvectors and angular momentum operator
πŸ‘︎ 2
πŸ’¬︎
πŸ‘€︎ u/almogg
πŸ“…︎ Jun 02 2011
🚨︎ report
Operation: Angular Momentum MK1 was somewhat successful. v.redd.it/4dcdcrzivqxz
πŸ‘︎ 7k
πŸ’¬︎
πŸ‘€︎ u/RoverRebellion
πŸ“…︎ Nov 13 2017
🚨︎ report
Angular momentum must be conserved v.redd.it/6f7ufundg3z71
πŸ‘︎ 3k
πŸ’¬︎
πŸ‘€︎ u/a_saddler
πŸ“…︎ Nov 12 2021
🚨︎ report
Angular momentum demo - Arms IN vs OUT v.redd.it/38nog119sj581
πŸ‘︎ 708
πŸ’¬︎
πŸ‘€︎ u/highnchillin_
πŸ“…︎ Dec 14 2021
🚨︎ report
Law of conservation of angular momentum! v.redd.it/3wvhd567xky71
πŸ‘︎ 1k
πŸ’¬︎
πŸ‘€︎ u/_marty_mcfly123_
πŸ“…︎ Nov 09 2021
🚨︎ report
Wind speed? Checked. Angular momentum? Checked. Merida is ready to launch
πŸ‘︎ 346
πŸ’¬︎
πŸ‘€︎ u/javier_aeoa
πŸ“…︎ Dec 30 2021
🚨︎ report
(+37699) Angular Momentum
πŸ‘︎ 33
πŸ’¬︎
πŸ‘€︎ u/edditbot
πŸ“…︎ Jan 06 2022
🚨︎ report
Angular momentum v.redd.it/1jtedx5omcw71
πŸ‘︎ 2k
πŸ’¬︎
πŸ‘€︎ u/cenabollywood
πŸ“…︎ Oct 29 2021
🚨︎ report
Angular momentum demo - Arms IN vs OUT v.redd.it/38nog119sj581
πŸ‘︎ 212
πŸ’¬︎
πŸ‘€︎ u/highnchillin_
πŸ“…︎ Dec 14 2021
🚨︎ report
[Quantum Mechanics] Less of a HW question and more of a general one: does the orbital angular momentum quantum number (L) span from 0 to n-1 (in general)?

Asking because upon review of my notes for the derivation of the hydrogen atom’s spectrum, it looks as though the considerations taken along the way to derive the spectrum (of hydrogen), namely the reasonings stemming from expressing the radial functions in terms of power series, are ultimately what fixed this upper limit of L to what I see it generally referenced as, n-1.

If this result is general, can someone explain why, atleast qualitatively? Thanks in advance.

πŸ‘︎ 10
πŸ’¬︎
πŸ‘€︎ u/Oni-Chan-Sama
πŸ“…︎ Dec 12 2021
🚨︎ report
The way he stretches his leg and pulls it back in, conserving angular momentum v.redd.it/zoez9dkpwpb81
πŸ‘︎ 14
πŸ’¬︎
πŸ‘€︎ u/nomaddd79
πŸ“…︎ Jan 15 2022
🚨︎ report
The black hole spin parameter is cJ/GM^2. That seems to be angular momentum per unit mass, but word on the street is that it's actually a unitless ratio. What does it represent?

thank you!

πŸ‘︎ 4
πŸ’¬︎
πŸ‘€︎ u/Vapes_THC_all_day
πŸ“…︎ Jan 20 2022
🚨︎ report
Angular momentum demo - Arms IN vs OUT v.redd.it/38nog119sj581
πŸ‘︎ 174
πŸ’¬︎
πŸ‘€︎ u/highnchillin_
πŸ“…︎ Dec 14 2021
🚨︎ report
Angular momentum/ reaction wheels in architecture?

Seeing the varying kinetic elements of buildings (with attention to things like pendulums) I got curious if reaction wheels have ever been used on land based structures.

Does anyone know of any applications?

Are rotating body type physics ever applied to building design?

πŸ‘︎ 3
πŸ’¬︎
πŸ“…︎ Jan 08 2022
🚨︎ report
Angular momentum v.redd.it/zia9j6oa39w71
πŸ‘︎ 1k
πŸ’¬︎
πŸ‘€︎ u/Templar_Ace
πŸ“…︎ Oct 29 2021
🚨︎ report
Does it make sense to wonder about the angular momentum of the entire universe?

Angular momentum is conserved, so there must be some quantity of angular momentum for the entire universe that never changes. What is the significance of this, if any?

πŸ‘︎ 18
πŸ’¬︎
πŸ‘€︎ u/GoGoGadgetDick95
πŸ“…︎ Nov 24 2021
🚨︎ report
The conservation of angular momentum never fails to amaze me. v.redd.it/mlnzmh2ndsv71
πŸ‘︎ 533
πŸ’¬︎
πŸ“…︎ Oct 26 2021
🚨︎ report
What's the precessional angular momentum of a Beyblade
πŸ‘︎ 57
πŸ’¬︎
πŸ“…︎ Nov 11 2021
🚨︎ report
A room temperature on-chip orbital angular momentum source that emits well collimated single photons has been demonstrated
πŸ‘︎ 3
πŸ’¬︎
πŸ‘€︎ u/Dr_Singularity
πŸ“…︎ Jan 13 2022
🚨︎ report
What would be the length of a day on a sphere of Uranium that had the same mass and angular momentum as Earth?
πŸ‘︎ 2
πŸ’¬︎
πŸ‘€︎ u/No_Acadia_9335
πŸ“…︎ Dec 15 2021
🚨︎ report
Angular momentum, it makes the world go round.
πŸ‘︎ 46
πŸ’¬︎
πŸ‘€︎ u/Zinabas
πŸ“…︎ Jan 18 2022
🚨︎ report

Please note that this site uses cookies to personalise content and adverts, to provide social media features, and to analyse web traffic. Click here for more information.